Evaluating $lim_{nrightarrowinfty} n[(1+frac{1}{n})^{n+1} - e]$












3












$begingroup$


How can I show, preferably with elementary methods, that $$lim_{nrightarrowinfty} nleft[left(1+frac{1}{n}right)^{n+1} - eright] = frac{e}{2}$$? All that would suffice for me is to show that the limit exists and is not negative. I've tried toying with binomial expansion but it didn't amount to anything unfortunately.










share|cite|improve this question











$endgroup$








  • 2




    $begingroup$
    Are you sure the answer is $e/2$ instead of $e$?
    $endgroup$
    – Larry
    Dec 26 '18 at 22:32










  • $begingroup$
    That's what Wolfram Alpha suggests. I'm not an expert so I don't doubt Wolfram's answers for problems at my level - if it's wrong, sorry for implying that. link to Wolfram
    $endgroup$
    – Bag of Chips
    Dec 26 '18 at 22:34












  • $begingroup$
    I see, wolfram alpha indeed gives e/2
    $endgroup$
    – Larry
    Dec 26 '18 at 22:36






  • 1




    $begingroup$
    I am getting this... $$lim_{n to infty}nbigg(bigg(1+frac{1}{n}bigg)^{n+1}-ebigg)$$ $$=lim_{n to infty}nbigg(bigg(1+frac{1}{n}bigg)^nbigg(1+frac{1}{n}bigg)-ebigg)$$ $$=lim_{n to infty}nbigg(ebigg(1+frac{1}{n}bigg)-ebigg)$$ $$=lim_{n to infty}nbigg(ebigg(1+frac{1}{n}-1bigg)bigg)$$ $$=lim_{n to infty}n(frac{e}{n})$$ $$=e$$
    $endgroup$
    – Rakibul Islam Prince
    Dec 26 '18 at 23:01






  • 1




    $begingroup$
    @RakibulIslamPrince: you cannot replace a part of the expression with its limit in general.
    $endgroup$
    – Paramanand Singh
    Dec 27 '18 at 0:31
















3












$begingroup$


How can I show, preferably with elementary methods, that $$lim_{nrightarrowinfty} nleft[left(1+frac{1}{n}right)^{n+1} - eright] = frac{e}{2}$$? All that would suffice for me is to show that the limit exists and is not negative. I've tried toying with binomial expansion but it didn't amount to anything unfortunately.










share|cite|improve this question











$endgroup$








  • 2




    $begingroup$
    Are you sure the answer is $e/2$ instead of $e$?
    $endgroup$
    – Larry
    Dec 26 '18 at 22:32










  • $begingroup$
    That's what Wolfram Alpha suggests. I'm not an expert so I don't doubt Wolfram's answers for problems at my level - if it's wrong, sorry for implying that. link to Wolfram
    $endgroup$
    – Bag of Chips
    Dec 26 '18 at 22:34












  • $begingroup$
    I see, wolfram alpha indeed gives e/2
    $endgroup$
    – Larry
    Dec 26 '18 at 22:36






  • 1




    $begingroup$
    I am getting this... $$lim_{n to infty}nbigg(bigg(1+frac{1}{n}bigg)^{n+1}-ebigg)$$ $$=lim_{n to infty}nbigg(bigg(1+frac{1}{n}bigg)^nbigg(1+frac{1}{n}bigg)-ebigg)$$ $$=lim_{n to infty}nbigg(ebigg(1+frac{1}{n}bigg)-ebigg)$$ $$=lim_{n to infty}nbigg(ebigg(1+frac{1}{n}-1bigg)bigg)$$ $$=lim_{n to infty}n(frac{e}{n})$$ $$=e$$
    $endgroup$
    – Rakibul Islam Prince
    Dec 26 '18 at 23:01






  • 1




    $begingroup$
    @RakibulIslamPrince: you cannot replace a part of the expression with its limit in general.
    $endgroup$
    – Paramanand Singh
    Dec 27 '18 at 0:31














3












3








3


1



$begingroup$


How can I show, preferably with elementary methods, that $$lim_{nrightarrowinfty} nleft[left(1+frac{1}{n}right)^{n+1} - eright] = frac{e}{2}$$? All that would suffice for me is to show that the limit exists and is not negative. I've tried toying with binomial expansion but it didn't amount to anything unfortunately.










share|cite|improve this question











$endgroup$




How can I show, preferably with elementary methods, that $$lim_{nrightarrowinfty} nleft[left(1+frac{1}{n}right)^{n+1} - eright] = frac{e}{2}$$? All that would suffice for me is to show that the limit exists and is not negative. I've tried toying with binomial expansion but it didn't amount to anything unfortunately.







calculus limits






share|cite|improve this question















share|cite|improve this question













share|cite|improve this question




share|cite|improve this question








edited Dec 26 '18 at 22:30









Larry

2,54531131




2,54531131










asked Dec 26 '18 at 22:19









Bag of ChipsBag of Chips

1187




1187








  • 2




    $begingroup$
    Are you sure the answer is $e/2$ instead of $e$?
    $endgroup$
    – Larry
    Dec 26 '18 at 22:32










  • $begingroup$
    That's what Wolfram Alpha suggests. I'm not an expert so I don't doubt Wolfram's answers for problems at my level - if it's wrong, sorry for implying that. link to Wolfram
    $endgroup$
    – Bag of Chips
    Dec 26 '18 at 22:34












  • $begingroup$
    I see, wolfram alpha indeed gives e/2
    $endgroup$
    – Larry
    Dec 26 '18 at 22:36






  • 1




    $begingroup$
    I am getting this... $$lim_{n to infty}nbigg(bigg(1+frac{1}{n}bigg)^{n+1}-ebigg)$$ $$=lim_{n to infty}nbigg(bigg(1+frac{1}{n}bigg)^nbigg(1+frac{1}{n}bigg)-ebigg)$$ $$=lim_{n to infty}nbigg(ebigg(1+frac{1}{n}bigg)-ebigg)$$ $$=lim_{n to infty}nbigg(ebigg(1+frac{1}{n}-1bigg)bigg)$$ $$=lim_{n to infty}n(frac{e}{n})$$ $$=e$$
    $endgroup$
    – Rakibul Islam Prince
    Dec 26 '18 at 23:01






  • 1




    $begingroup$
    @RakibulIslamPrince: you cannot replace a part of the expression with its limit in general.
    $endgroup$
    – Paramanand Singh
    Dec 27 '18 at 0:31














  • 2




    $begingroup$
    Are you sure the answer is $e/2$ instead of $e$?
    $endgroup$
    – Larry
    Dec 26 '18 at 22:32










  • $begingroup$
    That's what Wolfram Alpha suggests. I'm not an expert so I don't doubt Wolfram's answers for problems at my level - if it's wrong, sorry for implying that. link to Wolfram
    $endgroup$
    – Bag of Chips
    Dec 26 '18 at 22:34












  • $begingroup$
    I see, wolfram alpha indeed gives e/2
    $endgroup$
    – Larry
    Dec 26 '18 at 22:36






  • 1




    $begingroup$
    I am getting this... $$lim_{n to infty}nbigg(bigg(1+frac{1}{n}bigg)^{n+1}-ebigg)$$ $$=lim_{n to infty}nbigg(bigg(1+frac{1}{n}bigg)^nbigg(1+frac{1}{n}bigg)-ebigg)$$ $$=lim_{n to infty}nbigg(ebigg(1+frac{1}{n}bigg)-ebigg)$$ $$=lim_{n to infty}nbigg(ebigg(1+frac{1}{n}-1bigg)bigg)$$ $$=lim_{n to infty}n(frac{e}{n})$$ $$=e$$
    $endgroup$
    – Rakibul Islam Prince
    Dec 26 '18 at 23:01






  • 1




    $begingroup$
    @RakibulIslamPrince: you cannot replace a part of the expression with its limit in general.
    $endgroup$
    – Paramanand Singh
    Dec 27 '18 at 0:31








2




2




$begingroup$
Are you sure the answer is $e/2$ instead of $e$?
$endgroup$
– Larry
Dec 26 '18 at 22:32




$begingroup$
Are you sure the answer is $e/2$ instead of $e$?
$endgroup$
– Larry
Dec 26 '18 at 22:32












$begingroup$
That's what Wolfram Alpha suggests. I'm not an expert so I don't doubt Wolfram's answers for problems at my level - if it's wrong, sorry for implying that. link to Wolfram
$endgroup$
– Bag of Chips
Dec 26 '18 at 22:34






$begingroup$
That's what Wolfram Alpha suggests. I'm not an expert so I don't doubt Wolfram's answers for problems at my level - if it's wrong, sorry for implying that. link to Wolfram
$endgroup$
– Bag of Chips
Dec 26 '18 at 22:34














$begingroup$
I see, wolfram alpha indeed gives e/2
$endgroup$
– Larry
Dec 26 '18 at 22:36




$begingroup$
I see, wolfram alpha indeed gives e/2
$endgroup$
– Larry
Dec 26 '18 at 22:36




1




1




$begingroup$
I am getting this... $$lim_{n to infty}nbigg(bigg(1+frac{1}{n}bigg)^{n+1}-ebigg)$$ $$=lim_{n to infty}nbigg(bigg(1+frac{1}{n}bigg)^nbigg(1+frac{1}{n}bigg)-ebigg)$$ $$=lim_{n to infty}nbigg(ebigg(1+frac{1}{n}bigg)-ebigg)$$ $$=lim_{n to infty}nbigg(ebigg(1+frac{1}{n}-1bigg)bigg)$$ $$=lim_{n to infty}n(frac{e}{n})$$ $$=e$$
$endgroup$
– Rakibul Islam Prince
Dec 26 '18 at 23:01




$begingroup$
I am getting this... $$lim_{n to infty}nbigg(bigg(1+frac{1}{n}bigg)^{n+1}-ebigg)$$ $$=lim_{n to infty}nbigg(bigg(1+frac{1}{n}bigg)^nbigg(1+frac{1}{n}bigg)-ebigg)$$ $$=lim_{n to infty}nbigg(ebigg(1+frac{1}{n}bigg)-ebigg)$$ $$=lim_{n to infty}nbigg(ebigg(1+frac{1}{n}-1bigg)bigg)$$ $$=lim_{n to infty}n(frac{e}{n})$$ $$=e$$
$endgroup$
– Rakibul Islam Prince
Dec 26 '18 at 23:01




1




1




$begingroup$
@RakibulIslamPrince: you cannot replace a part of the expression with its limit in general.
$endgroup$
– Paramanand Singh
Dec 27 '18 at 0:31




$begingroup$
@RakibulIslamPrince: you cannot replace a part of the expression with its limit in general.
$endgroup$
– Paramanand Singh
Dec 27 '18 at 0:31










5 Answers
5






active

oldest

votes


















5












$begingroup$

With usual Taylor expansions calculate:




$nleft((1+frac 1n)^{n+1}-eright)
\=nleft(exp((n+1)ln(1+frac 1n))-eright)
\=nleft(exp((n+1)(frac 1n-frac 1{2n^2}+o(frac 1{n^2})))-eright)\=nleft(exp(frac 1n-underbrace{frac 1{2n^2}}_{(*)=0}+1-frac 1{2n}+o(frac 1n)))-eright)
\=nleft(exp(1+frac 1{2n}+o(frac 1n)))-eright)
\require{cancel}=nleft(cancel{e}+frac e{2n}+o(frac 1n)-cancel{e}right)
\=frac e{2}+o(1)to frac e2
$



(*) this term is too small for the resulting $o(frac 1n)$ thus it is simply ignored in this expansion.





About the comment of Rakibul Islam Prince:



What is wrong is that you take the limit of $(1+frac 1n)^n$ inside the calculation. Have you noticed the limit operator is exterior.



You cannot take partial limits as you wish.



In fact doing this is equivalent to ignoring the term $-frac 1{2n}$ coming from order $2$ in log expansion in my calculation.



In essence if you take the limit inside this would give
$$left(1+frac 1nright)^nleft(1+frac 1nright)-e=(e)(1)-e=0$$



Notice the second $1+frac 1n$ actually cannot stand as is.



Indeed, the correct calculation of expanding the logarithm to first order only would give:



$nleft(exp((n+1)(frac 1n+o(frac 1{n})))-eright)
\=nleft(exp(1+o(1))-eright)
\=n(e+o(1)-e)
\=0+o(n)$



which has not clear limit.






share|cite|improve this answer











$endgroup$













  • $begingroup$
    The question is "why can't I take the partial limit"?
    $endgroup$
    – Rakibul Islam Prince
    Dec 27 '18 at 11:01










  • $begingroup$
    @RakibulIslamPrince: think why can't you write $sqrt{a+b} =sqrt{a} +sqrt{b} $.
    $endgroup$
    – Paramanand Singh
    Dec 28 '18 at 3:31



















2












$begingroup$

Another (simple) solution involves applying L’Hopital after using $x=frac{1}{n}$ to deduce that your limit is equivalent to $frac{((e^{(1+x)logfrac{1}{x}}-e)}{x} to frac{e}{2} $which after differentiating becomes equivalent to $frac{frac{x}{x+1}-log(1+x)}{x^2} to frac{1}{2}$, for which you apply L’Hopital again and it becomes easy to evaluate, But I assume L’Hopital isn’t really too elementary either? :)






share|cite|improve this answer











$endgroup$





















    2












    $begingroup$

    This is a modification of the answer by Sorin Tirc



    $$lim_{nrightarrowinfty} nleft[left(1+frac{1}{n}right)^{n+1} - eright] = lim_{nrightarrowinfty} frac{e^{(n+1) ln left(1+frac{1}{n}right)} - e^0}{frac{1}{n}}\= lim_{nrightarrowinfty} frac{e^{(n+1) ln left(1+frac{1}{n}right)} - e^1}{(n+1) ln left(1+frac{1}{n}right)-1}frac{(n+1) ln left(1+frac{1}{n}right)-1}{frac{1}{n}} $$



    Now, since
    $$lim_{x to 0} frac{e^x-e^1}{x-1}=e$$ by the definition of the derivative you get
    $$lim_{nrightarrowinfty} nleft[left(1+frac{1}{n}right)^{n+1} - eright] = lim_{nrightarrowinfty} e frac{(n+1) ln left(1+frac{1}{n}right)-1}{frac{1}{n}}= lim_{nrightarrowinfty} [(n+1) ln left(1+frac{1}{n}right)-1]$$
    which can easi;y be calculated with L'Hospital.






    share|cite|improve this answer











    $endgroup$





















      0












      $begingroup$

      Considering$$a_n=nleft(left(1+frac{1}{n}right)^{n+1}-eright) $$ Consider
      $$x=left(1+frac{1}{n}right)^{n+1}implies log(x)=(n+1)logleft(1+frac{1}{n}right)$$ So, using Taylor
      $$log(x)=(n+1)left(frac{1}{n}-frac{1}{2 n^2}+frac{1}{3
      n^3}+Oleft(frac{1}{n^4}right)right)=1+frac{1}{2 n}-frac{1}{6 n^2}+Oleft(frac{1}{n^3}right)$$



      Continue with Taylor
      $$x=e^{log(x)}=e+frac{e}{2 n}-frac{e}{24 n^2}+Oleft(frac{1}{n^3}right)$$ Back to $a_n$
      $$a_n=nleft(e+frac{e}{2 n}-frac{e}{24 n^2}+Oleft(frac{1}{n^3}right)-eright)=frac{e}{2}-frac{e}{24 n}+Oleft(frac{1}{n^2}right) $$ which shows the limit and also how it is approached.



      Try with $n=10$ (which is far away from infinity) and use your pocket calculator
      $$a_{10}=frac{285311670611}{10000000000}-10 eapprox 1.34835$$ while the approximation would give $$frac{e}{2}-frac{e}{240}=frac{119 e}{240}approx 1.34781$$






      share|cite|improve this answer









      $endgroup$





















        0












        $begingroup$

        Let's put $x=1/n$ so that $xto 0^{+}$ and the expression under limit can be written as $$frac{(1+x)^{1+(1/x)}-e}{x}$$ The numerator can be expressed as $$ecdotdfrac{expleft(dfrac{1+x}{x}log(1+x)-1right)-1}{dfrac{1+x}{x}log(1+x)-1} cdot left(dfrac{1+x}{x}log(1+x)-1right) $$ and the middle fraction tends to $1$ so that the desired limit is equal to the limit of $$ecdotfrac{(1+x)log(1+x)-x}{x^2}$$ which is same as the limit of $$eleft(frac{log(1+x)}{x}+frac{log(1+x)-x}{x^2}right)$$ The first fraction in parenthesis tends to $1$ while the second one tends to $-1/2$ (via an easy application of L'Hospital's Rule or Taylor series) so that the desired limit is $e/2$.






        share|cite|improve this answer









        $endgroup$














          Your Answer








          StackExchange.ready(function() {
          var channelOptions = {
          tags: "".split(" "),
          id: "69"
          };
          initTagRenderer("".split(" "), "".split(" "), channelOptions);

          StackExchange.using("externalEditor", function() {
          // Have to fire editor after snippets, if snippets enabled
          if (StackExchange.settings.snippets.snippetsEnabled) {
          StackExchange.using("snippets", function() {
          createEditor();
          });
          }
          else {
          createEditor();
          }
          });

          function createEditor() {
          StackExchange.prepareEditor({
          heartbeatType: 'answer',
          autoActivateHeartbeat: false,
          convertImagesToLinks: true,
          noModals: true,
          showLowRepImageUploadWarning: true,
          reputationToPostImages: 10,
          bindNavPrevention: true,
          postfix: "",
          imageUploader: {
          brandingHtml: "Powered by u003ca class="icon-imgur-white" href="https://imgur.com/"u003eu003c/au003e",
          contentPolicyHtml: "User contributions licensed under u003ca href="https://creativecommons.org/licenses/by-sa/3.0/"u003ecc by-sa 3.0 with attribution requiredu003c/au003e u003ca href="https://stackoverflow.com/legal/content-policy"u003e(content policy)u003c/au003e",
          allowUrls: true
          },
          noCode: true, onDemand: true,
          discardSelector: ".discard-answer"
          ,immediatelyShowMarkdownHelp:true
          });


          }
          });














          draft saved

          draft discarded


















          StackExchange.ready(
          function () {
          StackExchange.openid.initPostLogin('.new-post-login', 'https%3a%2f%2fmath.stackexchange.com%2fquestions%2f3053366%2fevaluating-lim-n-rightarrow-infty-n1-frac1nn1-e%23new-answer', 'question_page');
          }
          );

          Post as a guest















          Required, but never shown

























          5 Answers
          5






          active

          oldest

          votes








          5 Answers
          5






          active

          oldest

          votes









          active

          oldest

          votes






          active

          oldest

          votes









          5












          $begingroup$

          With usual Taylor expansions calculate:




          $nleft((1+frac 1n)^{n+1}-eright)
          \=nleft(exp((n+1)ln(1+frac 1n))-eright)
          \=nleft(exp((n+1)(frac 1n-frac 1{2n^2}+o(frac 1{n^2})))-eright)\=nleft(exp(frac 1n-underbrace{frac 1{2n^2}}_{(*)=0}+1-frac 1{2n}+o(frac 1n)))-eright)
          \=nleft(exp(1+frac 1{2n}+o(frac 1n)))-eright)
          \require{cancel}=nleft(cancel{e}+frac e{2n}+o(frac 1n)-cancel{e}right)
          \=frac e{2}+o(1)to frac e2
          $



          (*) this term is too small for the resulting $o(frac 1n)$ thus it is simply ignored in this expansion.





          About the comment of Rakibul Islam Prince:



          What is wrong is that you take the limit of $(1+frac 1n)^n$ inside the calculation. Have you noticed the limit operator is exterior.



          You cannot take partial limits as you wish.



          In fact doing this is equivalent to ignoring the term $-frac 1{2n}$ coming from order $2$ in log expansion in my calculation.



          In essence if you take the limit inside this would give
          $$left(1+frac 1nright)^nleft(1+frac 1nright)-e=(e)(1)-e=0$$



          Notice the second $1+frac 1n$ actually cannot stand as is.



          Indeed, the correct calculation of expanding the logarithm to first order only would give:



          $nleft(exp((n+1)(frac 1n+o(frac 1{n})))-eright)
          \=nleft(exp(1+o(1))-eright)
          \=n(e+o(1)-e)
          \=0+o(n)$



          which has not clear limit.






          share|cite|improve this answer











          $endgroup$













          • $begingroup$
            The question is "why can't I take the partial limit"?
            $endgroup$
            – Rakibul Islam Prince
            Dec 27 '18 at 11:01










          • $begingroup$
            @RakibulIslamPrince: think why can't you write $sqrt{a+b} =sqrt{a} +sqrt{b} $.
            $endgroup$
            – Paramanand Singh
            Dec 28 '18 at 3:31
















          5












          $begingroup$

          With usual Taylor expansions calculate:




          $nleft((1+frac 1n)^{n+1}-eright)
          \=nleft(exp((n+1)ln(1+frac 1n))-eright)
          \=nleft(exp((n+1)(frac 1n-frac 1{2n^2}+o(frac 1{n^2})))-eright)\=nleft(exp(frac 1n-underbrace{frac 1{2n^2}}_{(*)=0}+1-frac 1{2n}+o(frac 1n)))-eright)
          \=nleft(exp(1+frac 1{2n}+o(frac 1n)))-eright)
          \require{cancel}=nleft(cancel{e}+frac e{2n}+o(frac 1n)-cancel{e}right)
          \=frac e{2}+o(1)to frac e2
          $



          (*) this term is too small for the resulting $o(frac 1n)$ thus it is simply ignored in this expansion.





          About the comment of Rakibul Islam Prince:



          What is wrong is that you take the limit of $(1+frac 1n)^n$ inside the calculation. Have you noticed the limit operator is exterior.



          You cannot take partial limits as you wish.



          In fact doing this is equivalent to ignoring the term $-frac 1{2n}$ coming from order $2$ in log expansion in my calculation.



          In essence if you take the limit inside this would give
          $$left(1+frac 1nright)^nleft(1+frac 1nright)-e=(e)(1)-e=0$$



          Notice the second $1+frac 1n$ actually cannot stand as is.



          Indeed, the correct calculation of expanding the logarithm to first order only would give:



          $nleft(exp((n+1)(frac 1n+o(frac 1{n})))-eright)
          \=nleft(exp(1+o(1))-eright)
          \=n(e+o(1)-e)
          \=0+o(n)$



          which has not clear limit.






          share|cite|improve this answer











          $endgroup$













          • $begingroup$
            The question is "why can't I take the partial limit"?
            $endgroup$
            – Rakibul Islam Prince
            Dec 27 '18 at 11:01










          • $begingroup$
            @RakibulIslamPrince: think why can't you write $sqrt{a+b} =sqrt{a} +sqrt{b} $.
            $endgroup$
            – Paramanand Singh
            Dec 28 '18 at 3:31














          5












          5








          5





          $begingroup$

          With usual Taylor expansions calculate:




          $nleft((1+frac 1n)^{n+1}-eright)
          \=nleft(exp((n+1)ln(1+frac 1n))-eright)
          \=nleft(exp((n+1)(frac 1n-frac 1{2n^2}+o(frac 1{n^2})))-eright)\=nleft(exp(frac 1n-underbrace{frac 1{2n^2}}_{(*)=0}+1-frac 1{2n}+o(frac 1n)))-eright)
          \=nleft(exp(1+frac 1{2n}+o(frac 1n)))-eright)
          \require{cancel}=nleft(cancel{e}+frac e{2n}+o(frac 1n)-cancel{e}right)
          \=frac e{2}+o(1)to frac e2
          $



          (*) this term is too small for the resulting $o(frac 1n)$ thus it is simply ignored in this expansion.





          About the comment of Rakibul Islam Prince:



          What is wrong is that you take the limit of $(1+frac 1n)^n$ inside the calculation. Have you noticed the limit operator is exterior.



          You cannot take partial limits as you wish.



          In fact doing this is equivalent to ignoring the term $-frac 1{2n}$ coming from order $2$ in log expansion in my calculation.



          In essence if you take the limit inside this would give
          $$left(1+frac 1nright)^nleft(1+frac 1nright)-e=(e)(1)-e=0$$



          Notice the second $1+frac 1n$ actually cannot stand as is.



          Indeed, the correct calculation of expanding the logarithm to first order only would give:



          $nleft(exp((n+1)(frac 1n+o(frac 1{n})))-eright)
          \=nleft(exp(1+o(1))-eright)
          \=n(e+o(1)-e)
          \=0+o(n)$



          which has not clear limit.






          share|cite|improve this answer











          $endgroup$



          With usual Taylor expansions calculate:




          $nleft((1+frac 1n)^{n+1}-eright)
          \=nleft(exp((n+1)ln(1+frac 1n))-eright)
          \=nleft(exp((n+1)(frac 1n-frac 1{2n^2}+o(frac 1{n^2})))-eright)\=nleft(exp(frac 1n-underbrace{frac 1{2n^2}}_{(*)=0}+1-frac 1{2n}+o(frac 1n)))-eright)
          \=nleft(exp(1+frac 1{2n}+o(frac 1n)))-eright)
          \require{cancel}=nleft(cancel{e}+frac e{2n}+o(frac 1n)-cancel{e}right)
          \=frac e{2}+o(1)to frac e2
          $



          (*) this term is too small for the resulting $o(frac 1n)$ thus it is simply ignored in this expansion.





          About the comment of Rakibul Islam Prince:



          What is wrong is that you take the limit of $(1+frac 1n)^n$ inside the calculation. Have you noticed the limit operator is exterior.



          You cannot take partial limits as you wish.



          In fact doing this is equivalent to ignoring the term $-frac 1{2n}$ coming from order $2$ in log expansion in my calculation.



          In essence if you take the limit inside this would give
          $$left(1+frac 1nright)^nleft(1+frac 1nright)-e=(e)(1)-e=0$$



          Notice the second $1+frac 1n$ actually cannot stand as is.



          Indeed, the correct calculation of expanding the logarithm to first order only would give:



          $nleft(exp((n+1)(frac 1n+o(frac 1{n})))-eright)
          \=nleft(exp(1+o(1))-eright)
          \=n(e+o(1)-e)
          \=0+o(n)$



          which has not clear limit.







          share|cite|improve this answer














          share|cite|improve this answer



          share|cite|improve this answer








          edited Dec 27 '18 at 0:13

























          answered Dec 26 '18 at 22:39









          zwimzwim

          12.7k832




          12.7k832












          • $begingroup$
            The question is "why can't I take the partial limit"?
            $endgroup$
            – Rakibul Islam Prince
            Dec 27 '18 at 11:01










          • $begingroup$
            @RakibulIslamPrince: think why can't you write $sqrt{a+b} =sqrt{a} +sqrt{b} $.
            $endgroup$
            – Paramanand Singh
            Dec 28 '18 at 3:31


















          • $begingroup$
            The question is "why can't I take the partial limit"?
            $endgroup$
            – Rakibul Islam Prince
            Dec 27 '18 at 11:01










          • $begingroup$
            @RakibulIslamPrince: think why can't you write $sqrt{a+b} =sqrt{a} +sqrt{b} $.
            $endgroup$
            – Paramanand Singh
            Dec 28 '18 at 3:31
















          $begingroup$
          The question is "why can't I take the partial limit"?
          $endgroup$
          – Rakibul Islam Prince
          Dec 27 '18 at 11:01




          $begingroup$
          The question is "why can't I take the partial limit"?
          $endgroup$
          – Rakibul Islam Prince
          Dec 27 '18 at 11:01












          $begingroup$
          @RakibulIslamPrince: think why can't you write $sqrt{a+b} =sqrt{a} +sqrt{b} $.
          $endgroup$
          – Paramanand Singh
          Dec 28 '18 at 3:31




          $begingroup$
          @RakibulIslamPrince: think why can't you write $sqrt{a+b} =sqrt{a} +sqrt{b} $.
          $endgroup$
          – Paramanand Singh
          Dec 28 '18 at 3:31











          2












          $begingroup$

          Another (simple) solution involves applying L’Hopital after using $x=frac{1}{n}$ to deduce that your limit is equivalent to $frac{((e^{(1+x)logfrac{1}{x}}-e)}{x} to frac{e}{2} $which after differentiating becomes equivalent to $frac{frac{x}{x+1}-log(1+x)}{x^2} to frac{1}{2}$, for which you apply L’Hopital again and it becomes easy to evaluate, But I assume L’Hopital isn’t really too elementary either? :)






          share|cite|improve this answer











          $endgroup$


















            2












            $begingroup$

            Another (simple) solution involves applying L’Hopital after using $x=frac{1}{n}$ to deduce that your limit is equivalent to $frac{((e^{(1+x)logfrac{1}{x}}-e)}{x} to frac{e}{2} $which after differentiating becomes equivalent to $frac{frac{x}{x+1}-log(1+x)}{x^2} to frac{1}{2}$, for which you apply L’Hopital again and it becomes easy to evaluate, But I assume L’Hopital isn’t really too elementary either? :)






            share|cite|improve this answer











            $endgroup$
















              2












              2








              2





              $begingroup$

              Another (simple) solution involves applying L’Hopital after using $x=frac{1}{n}$ to deduce that your limit is equivalent to $frac{((e^{(1+x)logfrac{1}{x}}-e)}{x} to frac{e}{2} $which after differentiating becomes equivalent to $frac{frac{x}{x+1}-log(1+x)}{x^2} to frac{1}{2}$, for which you apply L’Hopital again and it becomes easy to evaluate, But I assume L’Hopital isn’t really too elementary either? :)






              share|cite|improve this answer











              $endgroup$



              Another (simple) solution involves applying L’Hopital after using $x=frac{1}{n}$ to deduce that your limit is equivalent to $frac{((e^{(1+x)logfrac{1}{x}}-e)}{x} to frac{e}{2} $which after differentiating becomes equivalent to $frac{frac{x}{x+1}-log(1+x)}{x^2} to frac{1}{2}$, for which you apply L’Hopital again and it becomes easy to evaluate, But I assume L’Hopital isn’t really too elementary either? :)







              share|cite|improve this answer














              share|cite|improve this answer



              share|cite|improve this answer








              edited Dec 26 '18 at 22:53

























              answered Dec 26 '18 at 22:45









              Sorin TircSorin Tirc

              1,875213




              1,875213























                  2












                  $begingroup$

                  This is a modification of the answer by Sorin Tirc



                  $$lim_{nrightarrowinfty} nleft[left(1+frac{1}{n}right)^{n+1} - eright] = lim_{nrightarrowinfty} frac{e^{(n+1) ln left(1+frac{1}{n}right)} - e^0}{frac{1}{n}}\= lim_{nrightarrowinfty} frac{e^{(n+1) ln left(1+frac{1}{n}right)} - e^1}{(n+1) ln left(1+frac{1}{n}right)-1}frac{(n+1) ln left(1+frac{1}{n}right)-1}{frac{1}{n}} $$



                  Now, since
                  $$lim_{x to 0} frac{e^x-e^1}{x-1}=e$$ by the definition of the derivative you get
                  $$lim_{nrightarrowinfty} nleft[left(1+frac{1}{n}right)^{n+1} - eright] = lim_{nrightarrowinfty} e frac{(n+1) ln left(1+frac{1}{n}right)-1}{frac{1}{n}}= lim_{nrightarrowinfty} [(n+1) ln left(1+frac{1}{n}right)-1]$$
                  which can easi;y be calculated with L'Hospital.






                  share|cite|improve this answer











                  $endgroup$


















                    2












                    $begingroup$

                    This is a modification of the answer by Sorin Tirc



                    $$lim_{nrightarrowinfty} nleft[left(1+frac{1}{n}right)^{n+1} - eright] = lim_{nrightarrowinfty} frac{e^{(n+1) ln left(1+frac{1}{n}right)} - e^0}{frac{1}{n}}\= lim_{nrightarrowinfty} frac{e^{(n+1) ln left(1+frac{1}{n}right)} - e^1}{(n+1) ln left(1+frac{1}{n}right)-1}frac{(n+1) ln left(1+frac{1}{n}right)-1}{frac{1}{n}} $$



                    Now, since
                    $$lim_{x to 0} frac{e^x-e^1}{x-1}=e$$ by the definition of the derivative you get
                    $$lim_{nrightarrowinfty} nleft[left(1+frac{1}{n}right)^{n+1} - eright] = lim_{nrightarrowinfty} e frac{(n+1) ln left(1+frac{1}{n}right)-1}{frac{1}{n}}= lim_{nrightarrowinfty} [(n+1) ln left(1+frac{1}{n}right)-1]$$
                    which can easi;y be calculated with L'Hospital.






                    share|cite|improve this answer











                    $endgroup$
















                      2












                      2








                      2





                      $begingroup$

                      This is a modification of the answer by Sorin Tirc



                      $$lim_{nrightarrowinfty} nleft[left(1+frac{1}{n}right)^{n+1} - eright] = lim_{nrightarrowinfty} frac{e^{(n+1) ln left(1+frac{1}{n}right)} - e^0}{frac{1}{n}}\= lim_{nrightarrowinfty} frac{e^{(n+1) ln left(1+frac{1}{n}right)} - e^1}{(n+1) ln left(1+frac{1}{n}right)-1}frac{(n+1) ln left(1+frac{1}{n}right)-1}{frac{1}{n}} $$



                      Now, since
                      $$lim_{x to 0} frac{e^x-e^1}{x-1}=e$$ by the definition of the derivative you get
                      $$lim_{nrightarrowinfty} nleft[left(1+frac{1}{n}right)^{n+1} - eright] = lim_{nrightarrowinfty} e frac{(n+1) ln left(1+frac{1}{n}right)-1}{frac{1}{n}}= lim_{nrightarrowinfty} [(n+1) ln left(1+frac{1}{n}right)-1]$$
                      which can easi;y be calculated with L'Hospital.






                      share|cite|improve this answer











                      $endgroup$



                      This is a modification of the answer by Sorin Tirc



                      $$lim_{nrightarrowinfty} nleft[left(1+frac{1}{n}right)^{n+1} - eright] = lim_{nrightarrowinfty} frac{e^{(n+1) ln left(1+frac{1}{n}right)} - e^0}{frac{1}{n}}\= lim_{nrightarrowinfty} frac{e^{(n+1) ln left(1+frac{1}{n}right)} - e^1}{(n+1) ln left(1+frac{1}{n}right)-1}frac{(n+1) ln left(1+frac{1}{n}right)-1}{frac{1}{n}} $$



                      Now, since
                      $$lim_{x to 0} frac{e^x-e^1}{x-1}=e$$ by the definition of the derivative you get
                      $$lim_{nrightarrowinfty} nleft[left(1+frac{1}{n}right)^{n+1} - eright] = lim_{nrightarrowinfty} e frac{(n+1) ln left(1+frac{1}{n}right)-1}{frac{1}{n}}= lim_{nrightarrowinfty} [(n+1) ln left(1+frac{1}{n}right)-1]$$
                      which can easi;y be calculated with L'Hospital.







                      share|cite|improve this answer














                      share|cite|improve this answer



                      share|cite|improve this answer








                      edited Dec 27 '18 at 0:12









                      Botond

                      6,57531034




                      6,57531034










                      answered Dec 26 '18 at 22:55









                      N. S.N. S.

                      105k7115210




                      105k7115210























                          0












                          $begingroup$

                          Considering$$a_n=nleft(left(1+frac{1}{n}right)^{n+1}-eright) $$ Consider
                          $$x=left(1+frac{1}{n}right)^{n+1}implies log(x)=(n+1)logleft(1+frac{1}{n}right)$$ So, using Taylor
                          $$log(x)=(n+1)left(frac{1}{n}-frac{1}{2 n^2}+frac{1}{3
                          n^3}+Oleft(frac{1}{n^4}right)right)=1+frac{1}{2 n}-frac{1}{6 n^2}+Oleft(frac{1}{n^3}right)$$



                          Continue with Taylor
                          $$x=e^{log(x)}=e+frac{e}{2 n}-frac{e}{24 n^2}+Oleft(frac{1}{n^3}right)$$ Back to $a_n$
                          $$a_n=nleft(e+frac{e}{2 n}-frac{e}{24 n^2}+Oleft(frac{1}{n^3}right)-eright)=frac{e}{2}-frac{e}{24 n}+Oleft(frac{1}{n^2}right) $$ which shows the limit and also how it is approached.



                          Try with $n=10$ (which is far away from infinity) and use your pocket calculator
                          $$a_{10}=frac{285311670611}{10000000000}-10 eapprox 1.34835$$ while the approximation would give $$frac{e}{2}-frac{e}{240}=frac{119 e}{240}approx 1.34781$$






                          share|cite|improve this answer









                          $endgroup$


















                            0












                            $begingroup$

                            Considering$$a_n=nleft(left(1+frac{1}{n}right)^{n+1}-eright) $$ Consider
                            $$x=left(1+frac{1}{n}right)^{n+1}implies log(x)=(n+1)logleft(1+frac{1}{n}right)$$ So, using Taylor
                            $$log(x)=(n+1)left(frac{1}{n}-frac{1}{2 n^2}+frac{1}{3
                            n^3}+Oleft(frac{1}{n^4}right)right)=1+frac{1}{2 n}-frac{1}{6 n^2}+Oleft(frac{1}{n^3}right)$$



                            Continue with Taylor
                            $$x=e^{log(x)}=e+frac{e}{2 n}-frac{e}{24 n^2}+Oleft(frac{1}{n^3}right)$$ Back to $a_n$
                            $$a_n=nleft(e+frac{e}{2 n}-frac{e}{24 n^2}+Oleft(frac{1}{n^3}right)-eright)=frac{e}{2}-frac{e}{24 n}+Oleft(frac{1}{n^2}right) $$ which shows the limit and also how it is approached.



                            Try with $n=10$ (which is far away from infinity) and use your pocket calculator
                            $$a_{10}=frac{285311670611}{10000000000}-10 eapprox 1.34835$$ while the approximation would give $$frac{e}{2}-frac{e}{240}=frac{119 e}{240}approx 1.34781$$






                            share|cite|improve this answer









                            $endgroup$
















                              0












                              0








                              0





                              $begingroup$

                              Considering$$a_n=nleft(left(1+frac{1}{n}right)^{n+1}-eright) $$ Consider
                              $$x=left(1+frac{1}{n}right)^{n+1}implies log(x)=(n+1)logleft(1+frac{1}{n}right)$$ So, using Taylor
                              $$log(x)=(n+1)left(frac{1}{n}-frac{1}{2 n^2}+frac{1}{3
                              n^3}+Oleft(frac{1}{n^4}right)right)=1+frac{1}{2 n}-frac{1}{6 n^2}+Oleft(frac{1}{n^3}right)$$



                              Continue with Taylor
                              $$x=e^{log(x)}=e+frac{e}{2 n}-frac{e}{24 n^2}+Oleft(frac{1}{n^3}right)$$ Back to $a_n$
                              $$a_n=nleft(e+frac{e}{2 n}-frac{e}{24 n^2}+Oleft(frac{1}{n^3}right)-eright)=frac{e}{2}-frac{e}{24 n}+Oleft(frac{1}{n^2}right) $$ which shows the limit and also how it is approached.



                              Try with $n=10$ (which is far away from infinity) and use your pocket calculator
                              $$a_{10}=frac{285311670611}{10000000000}-10 eapprox 1.34835$$ while the approximation would give $$frac{e}{2}-frac{e}{240}=frac{119 e}{240}approx 1.34781$$






                              share|cite|improve this answer









                              $endgroup$



                              Considering$$a_n=nleft(left(1+frac{1}{n}right)^{n+1}-eright) $$ Consider
                              $$x=left(1+frac{1}{n}right)^{n+1}implies log(x)=(n+1)logleft(1+frac{1}{n}right)$$ So, using Taylor
                              $$log(x)=(n+1)left(frac{1}{n}-frac{1}{2 n^2}+frac{1}{3
                              n^3}+Oleft(frac{1}{n^4}right)right)=1+frac{1}{2 n}-frac{1}{6 n^2}+Oleft(frac{1}{n^3}right)$$



                              Continue with Taylor
                              $$x=e^{log(x)}=e+frac{e}{2 n}-frac{e}{24 n^2}+Oleft(frac{1}{n^3}right)$$ Back to $a_n$
                              $$a_n=nleft(e+frac{e}{2 n}-frac{e}{24 n^2}+Oleft(frac{1}{n^3}right)-eright)=frac{e}{2}-frac{e}{24 n}+Oleft(frac{1}{n^2}right) $$ which shows the limit and also how it is approached.



                              Try with $n=10$ (which is far away from infinity) and use your pocket calculator
                              $$a_{10}=frac{285311670611}{10000000000}-10 eapprox 1.34835$$ while the approximation would give $$frac{e}{2}-frac{e}{240}=frac{119 e}{240}approx 1.34781$$







                              share|cite|improve this answer












                              share|cite|improve this answer



                              share|cite|improve this answer










                              answered Dec 27 '18 at 4:45









                              Claude LeiboviciClaude Leibovici

                              125k1158135




                              125k1158135























                                  0












                                  $begingroup$

                                  Let's put $x=1/n$ so that $xto 0^{+}$ and the expression under limit can be written as $$frac{(1+x)^{1+(1/x)}-e}{x}$$ The numerator can be expressed as $$ecdotdfrac{expleft(dfrac{1+x}{x}log(1+x)-1right)-1}{dfrac{1+x}{x}log(1+x)-1} cdot left(dfrac{1+x}{x}log(1+x)-1right) $$ and the middle fraction tends to $1$ so that the desired limit is equal to the limit of $$ecdotfrac{(1+x)log(1+x)-x}{x^2}$$ which is same as the limit of $$eleft(frac{log(1+x)}{x}+frac{log(1+x)-x}{x^2}right)$$ The first fraction in parenthesis tends to $1$ while the second one tends to $-1/2$ (via an easy application of L'Hospital's Rule or Taylor series) so that the desired limit is $e/2$.






                                  share|cite|improve this answer









                                  $endgroup$


















                                    0












                                    $begingroup$

                                    Let's put $x=1/n$ so that $xto 0^{+}$ and the expression under limit can be written as $$frac{(1+x)^{1+(1/x)}-e}{x}$$ The numerator can be expressed as $$ecdotdfrac{expleft(dfrac{1+x}{x}log(1+x)-1right)-1}{dfrac{1+x}{x}log(1+x)-1} cdot left(dfrac{1+x}{x}log(1+x)-1right) $$ and the middle fraction tends to $1$ so that the desired limit is equal to the limit of $$ecdotfrac{(1+x)log(1+x)-x}{x^2}$$ which is same as the limit of $$eleft(frac{log(1+x)}{x}+frac{log(1+x)-x}{x^2}right)$$ The first fraction in parenthesis tends to $1$ while the second one tends to $-1/2$ (via an easy application of L'Hospital's Rule or Taylor series) so that the desired limit is $e/2$.






                                    share|cite|improve this answer









                                    $endgroup$
















                                      0












                                      0








                                      0





                                      $begingroup$

                                      Let's put $x=1/n$ so that $xto 0^{+}$ and the expression under limit can be written as $$frac{(1+x)^{1+(1/x)}-e}{x}$$ The numerator can be expressed as $$ecdotdfrac{expleft(dfrac{1+x}{x}log(1+x)-1right)-1}{dfrac{1+x}{x}log(1+x)-1} cdot left(dfrac{1+x}{x}log(1+x)-1right) $$ and the middle fraction tends to $1$ so that the desired limit is equal to the limit of $$ecdotfrac{(1+x)log(1+x)-x}{x^2}$$ which is same as the limit of $$eleft(frac{log(1+x)}{x}+frac{log(1+x)-x}{x^2}right)$$ The first fraction in parenthesis tends to $1$ while the second one tends to $-1/2$ (via an easy application of L'Hospital's Rule or Taylor series) so that the desired limit is $e/2$.






                                      share|cite|improve this answer









                                      $endgroup$



                                      Let's put $x=1/n$ so that $xto 0^{+}$ and the expression under limit can be written as $$frac{(1+x)^{1+(1/x)}-e}{x}$$ The numerator can be expressed as $$ecdotdfrac{expleft(dfrac{1+x}{x}log(1+x)-1right)-1}{dfrac{1+x}{x}log(1+x)-1} cdot left(dfrac{1+x}{x}log(1+x)-1right) $$ and the middle fraction tends to $1$ so that the desired limit is equal to the limit of $$ecdotfrac{(1+x)log(1+x)-x}{x^2}$$ which is same as the limit of $$eleft(frac{log(1+x)}{x}+frac{log(1+x)-x}{x^2}right)$$ The first fraction in parenthesis tends to $1$ while the second one tends to $-1/2$ (via an easy application of L'Hospital's Rule or Taylor series) so that the desired limit is $e/2$.







                                      share|cite|improve this answer












                                      share|cite|improve this answer



                                      share|cite|improve this answer










                                      answered Dec 27 '18 at 5:23









                                      Paramanand SinghParamanand Singh

                                      51.4k560170




                                      51.4k560170






























                                          draft saved

                                          draft discarded




















































                                          Thanks for contributing an answer to Mathematics Stack Exchange!


                                          • Please be sure to answer the question. Provide details and share your research!

                                          But avoid



                                          • Asking for help, clarification, or responding to other answers.

                                          • Making statements based on opinion; back them up with references or personal experience.


                                          Use MathJax to format equations. MathJax reference.


                                          To learn more, see our tips on writing great answers.




                                          draft saved


                                          draft discarded














                                          StackExchange.ready(
                                          function () {
                                          StackExchange.openid.initPostLogin('.new-post-login', 'https%3a%2f%2fmath.stackexchange.com%2fquestions%2f3053366%2fevaluating-lim-n-rightarrow-infty-n1-frac1nn1-e%23new-answer', 'question_page');
                                          }
                                          );

                                          Post as a guest















                                          Required, but never shown





















































                                          Required, but never shown














                                          Required, but never shown












                                          Required, but never shown







                                          Required, but never shown

































                                          Required, but never shown














                                          Required, but never shown












                                          Required, but never shown







                                          Required, but never shown







                                          Popular posts from this blog

                                          Le Mesnil-Réaume

                                          Ida-Boy-Ed-Garten

                                          web3.py web3.isConnected() returns false always